what is the gcf for 84 and 54

Answers

Answer 1
Your answer for the greatest common factor is 6

Related Questions

Simplify the expression: z8 * z-3

Answers

Answer:

z^5

Step-by-step explanation:

z^8  * z^-3

Since we are multiplying exponents and the bases are the same, we can add the exponents

z^(8-3)

z^5

Identify the error in each problem. Explain each error and provide the correction solution. Use at least two complete sentences. PLEASE SOMEONE HELP QUICK ITS DUE TOMORROW AND I WILL MARK YOU BRAINLIEST

Answers

Answer:

the mistake is the addition sign. When a number is next to a parenthesis it means multiplication

3/4 x - 1/2 - 4 = 12

Answers

Answer: x=22

Step-by-step explanation:

Step 1: Simplify both sides of the equation.

Step 2: Add 9/2 to both sides.

Step 3: Multiply both sides by 4/3.

Let n be a whole number, and consider the statements below. p: n is a multiple of two. q: n is an even number. Which of the following is equivalent to ~q → ~p? A. ~q → ~p B. q → p C. p → q D. ~p →~

Answers

Answer:

C. p -> q

Step-by-step explanation:

Just did this on Edge2020. Hope this helps :)

help please will mark brainliest

Answers

Answer:

i can't see that

Step-by-step explanation:

plzz sent me a clear photo

Answer:

1. 1

2. 1  ⋅  10 ^24

3. 1000000000

4. 1

5. 1  ⋅  10 ^20

6. 1  ⋅  10 ^30

7. 1  ⋅  10 ^27

8. 10000

9. 0.01

10. 0.0001

11. 0.1

Hopefully these are right, I simplified all of the problems...sorry if they are wrong

Step-by-step explanation:

Christine wants to buy strawberries and raspberries to bring to a party. Strawberries cost $1.65 per pound and raspberries cost $2.25 per pound. She only has $15 to spend on berries. Which inequality represents the situation where she buys x pounds of strawberries and y pounds of raspberries?

Answers

Answer:

1.65x + 2.25y ≤ 15

Step-by-step explanation:

Strawberries = $1.65 per pound Raspberries = $2.25 per pound

Amount with Christine = $15

Let

pounds of strawberries = x

pounds of raspberries = y

PxQx + PyQy ≤ 15

1.65x + 2.25y ≤ 15

The inequality which represent the situation where she buys x pounds of strawberries and y pounds of raspberries is 1.65x + 2.25y ≤ 15

1.65x + 2.25y less than or equal to 15

Which expression has the same value as -18-(-9)?
0 - 18+2
-12-(-3)
- 1945
O-8-(-4)

Answers

Answer:

the answer is -12-(-3)

Step-by-step explanation:

The expression equivalent to -18-(-9) is -12-(-3)

What are expressions?

An expression in maths is a sentence with a minimum of two numbers or variables and at least one maths operation.

Given is an expression, -18-(-9), we are given to find the expression equivalent to it,

-18-(-9) = -18+9 = -9

1) -18+2 = -16 (not equivalent)

2) -12-(-3) = -12+3 = -9

Therefore, we see, the value of expressions -12-(-3) and -18-(-9) are equivalent,

Hence, the expression equivalent to -18-(-9) is -12-(-3)

For more references on expressions, click;

https://brainly.com/question/14083225

#SPJ2

Factorise
a) 2ax + 6ay​

Answers

Answer:

2a(x + 3y)

Step-by-step explanation:

2ax + 6ay

factor out a 2a

2a(x + 3y)

From the expression we can understand that 2a divides into 2ax and 6ay. Therefore, the answer is:
2a(x + 3y)

meaning of cube root

Answers

Answer:

The cube root of a number is a special value that, when used in a multiplication three times, gives that number. Example: 3 × 3 × 3 = 27, so the cube root of 27 is 3. See: Square Root. Cubes and Cube Roots.

NEED HELP ASAP
given m||n find the value of x

Answers

The alternate interior angle theorem states that when two parallel lines are cut by a transversal, the resulting alternate interior angles are congruent.

This means that [tex]6x-8=5x+14[/tex]

[tex]x=22[/tex]

Hope this helps. 頑張って!

27x + 45y and 7(9x + 7y) are these equivalent?

Answers

Answer:

No

Step-by-step explanation:

If you want to know whether they are equivalent or no, we will first have to find the value of both.

[tex]27x+45y[/tex]

Since there aren't any like terms here, your answer would be

[tex]27x+45y[/tex]

Let's solve for next one.

[tex]7(9x+7y)[/tex]

Let's add in parenthesis to 7.

[tex](7)(9x+7y)[/tex]

Now we will separate everything. And also you will have to add.

[tex](7)(9x)+(7)(7y)[/tex]

Now that we added we got our answer.

[tex]63x+49y[/tex]

Both values were not the same, so therefore, your answer is no, they are not equivalent.

Hope this helps!

No

Step-by-step explanation:

If you want to know whether they are equivalent or no, we will first have to find the value of both.

Since there aren't any like terms here, your answer would be

Let's solve for next one.

Let's add in parenthesis to 7.

Now we will separate everything. And also you will have to add.

Now that we added we got our answer.

Both values were not the same, so therefore, your answer is no, they are not equivalent.

Hope this helps!

Read more on Brainly.com - https://brainly.com/question/17379473#readmore

find 3+root 2/3-root 2=a+b root2

Answers

Answer:

a = [tex]\frac{11}{7}[/tex] ; b = [tex]\frac{6}{7}[/tex]

Step-by-step explanation:

[tex]\frac{3 + \sqrt{2}}{3 - \sqrt{2} } = a + b\sqrt{2} \\\\[/tex]

Rationalising [tex]\frac{3 + \sqrt{2}}{3 - \sqrt{2} }[/tex] gives :-

[tex]\frac{3 + \sqrt{2}}{3 - \sqrt{2} } = \frac{(3 + \sqrt{2})(3 + \sqrt{2})}{(3 - \sqrt{2})(3 + \sqrt{2}) } = \frac{(3 + \sqrt{2})^2}{3^2 - (\sqrt{2})^2 } = \frac{11 +6\sqrt{2} }{7}[/tex]

Comparing [tex]\frac{11 + 6\sqrt{2} }{7}[/tex] with [tex]a + b\sqrt{2}[/tex] gives

a = [tex]\frac{11}{7}[/tex] & b = [tex]\frac{6}{7}[/tex]

Answer:

[tex]\frac{11}{7}[/tex] + [tex]\frac{6}{7}[/tex] [tex]\sqrt{2}[/tex]

Step-by-step explanation:

Given

[tex]\frac{3+\sqrt{2} }{3-\sqrt{2} }[/tex]

Multiply the numerator/ denominator by the conjugate of the denominator.

The conjugate of 3 - [tex]\sqrt{2}[/tex] is 3 + [tex]\sqrt{2}[/tex] , thus

= [tex]\frac{(3+\sqrt{2})(3+\sqrt{2}) }{(3-\sqrt{2})(3+\sqrt{2}) }[/tex] ← expand numerator/ denominator using FOIL

= [tex]\frac{9+6\sqrt{2}+2 }{9-2}[/tex]

= [tex]\frac{11+6\sqrt{2} }{7}[/tex]

= [tex]\frac{11}{7}[/tex] + [tex]\frac{6}{7}[/tex] [tex]\sqrt{2}[/tex] ← in the form a + b[tex]\sqrt{2}[/tex]

with a = [tex]\frac{11}{7}[/tex] and b = [tex]\frac{6}{7}[/tex]

How much of the circle is shaded? simply your answer

Answers

Answer:

The part that is shaded is 5/14

Step-by-step explanation:

The total circle is 1

1/2 + 1/7 + x = 1  where x is the shaded area

Multiply each side by 14 to get rid of the fractions

14(1/2 + 1/7 + x) = 1*14

7 + 2 + 14x = 14

9+14x = 14

Subtract 9 from each side

14x = 5

Divide by 14

x = 5/14

The part that is shaded is 5/14

i need help its urgent its due today geometry

Answers

Rays, vertex, the vertex of the angle,BXC

I REALLY NEED HELP PLEASE HELP ME

Answers

Answer:

52^8

Step-by-step explanation:

26^8 = 208827064576 + 26

One year, Mr James travels 8 x 10^3 miles for his job. The next year he traveled 1 x 10^4 miles. How many more miles did he travel the second year than he did the first year? Please give me the answer and explain ToT

Answers

Answer:

2 000 miles

Step-by-step explanation:

In the first year, Mr James travels 8 x [tex]10^{3}[/tex] miles = 8 000 miles.

The second year, he traveled 1 x [tex]10^{4}[/tex] miles = 10 000 miles.

Comparing the distance covered in the two years,

10 000 miles - 8 000 miles = 2 000 miles

Therefore, Mr James travels 2 000 more miles in the second year than he did the first year.

Let's assume that Mr James conveys himself to his place of work all through the first year in his private car. But in the second year, there were some days in which the car was not available. So he had to go for his job by public transportation. This itch during those days, could cause an increase in the distance covered from his home to his place of work for the second year.

This year, Zachary has been babysitting his young cousins after school for $70 a month. His uncle also
gave him an extra bonus of $100 for his excellent work. Since school started, Zachary has earned more
than $500. How many months ago did school start?

Answers

Answer:

8

Step-by-step explanation:

if f(x)=4-3x*2 then f(-2) is

Answers

Answer:

16

Step-by-step explanation:

f(-2)=4-3(-2)*2

f(-2)=4-3(-4)

f(-2)=4+12

f(-2)=16

The area of a sector is 30 m2 in a circle with radius 4 m. What is the arc length of the sector?

Answers

Answer:

15 m

Step-by-step explanation:

The area of the circle is

A =pi r^2

A = pi 4^2 = 16 pi

The area of the sector is 30

The fraction is

30/16 pi  

Take this time 2pi which are the  radians of a circle

30 /16 pi * 2 pi = 15/4

This is the number of radians the angle is

The arc length is s = r * theta where theta is in radians

s = r  theta

  = 4 * 15/4

  = 15

Jodi is considering taking online classes at two websites, LearnCenter and EduWorld. Each site requires that students pay a base membership fee, plus they charge a rate per class. The two graphs show the cost for taking online classes through the two websites. A graph titled Cost to Take Classes at LearnCenter has number of classes on the x-axis and cost in dollars on the y-axis. A line goes through points (0, 50) and (1, 70). A graph titled Cost to Take Classes at EduWorld has number of classes on the x-axis and cost in dollars on the y-axis. A line goes through points (0, 20) and (1, 60).

Answers

Answer:

B-LearnCenter charges a higher membership fee but a lower rate per class.

Step-by-step explanation:

Answer:B-learning for ever

Step-by-step explanation:

solve for x (2x+1) (x+29)​

Answers

The answer would be 2x^2+59x+29


Need help on 3 thank you

Answers

Answer:

Cost: $10.00

Number of Tickets:26

Step-by-step explanation:

Each ticket is $0.50

20*$0.50=$10.00

For the tickets:

$13.00/$0.50=26 tickets

The following data points represent the volume of gas in each race car driver's tank (in liters). Sort the data from least to greatest. Find the interquartile range (IQR) of the data set. \text{L}Lstart text, L, end text \text{L}Lstart text, L, end text

Answers

Answer:

its \text{L}Lstart text, L, end text

Step-by-step explanation:

The sorting of the dataset and the information from the five number summary, which includes the three quartiles and the minimum and maximum values indicates that we get;

1. 2.8, 4, 4.3, 6, 7.5, 8.5, 9, 11.6, 12, 12.1

2. The interquartile range is; 7.745

What is the five number summary of interquartile range of a data set?

The five number summary includes the first, second and third quartiles, Q₁, Q₂, and Q₃, and maximum and minimum values of the dataset, which serves to describe the characteristics of the data.

here, we have,

1. The sorted data from least to greatest can be presented as follows;

2.8, 4, 4.3, 6, 7.5, 8.5, 9, 11.6, 12, 12.1

Therefore, the original data in the question is already sorted from least to the greatest

2. The interquartile range, IQR, is the the difference between the third quartile, Q₃, and the first quartiles, Q₁, of the data.

IQR = Q₃ - Q₁

The dataset indicates that we get;

The number of datapoints in the dataset, n = 10

The first quartile = The (N + 1)/4 percentile value

First quartile, Q₁ = (10 + 1)/4 × 10 = The 2.75th value

The 2.75th value is the value 0.75 = 3/4 larger than the 2nd value, which can be found as follows;

Q₁ = The 2.75th value = 4 + (4.3 - 4) × (3/4)  = 4.225

The third quartile, Q₃ = The (N + 1) × (3/40th value, therefore;

Q₃ = The (10 + 1) × (3/4)th value = The 8.25th value (The value 0.25th more than the 8th value )

Q₃ = 11.6 + (12 - 11.6) × 0.25 = 11.7

The interquartile range, IQR = 11.7 - 4.225 = 7.475

The interquartile range = 7.475

Learn more on the interquartile range of a dataset here: brainly.com/question/13119591

#SPJ7

Complete question:

The following data points represent the volume of gas in each race car driver's tank (in liters).

Answer 2 questions about the data set.

1. Sort the data from least to greatest.

2.8

4

4.3

6

7.5

8.5

9

11.6

12

12.1

2. Find the interquartile range (IQR) of the data set.

L

What is the solution to this equation?
X-9 = 17
O A. x = 28
O B. x = 12
O c. x = 8
O D. x = 26

Answers

Answer:

Step-by-step explanation:

x -  9 = 17

x = 26

the answer is D

Find the factors of the following numbers.
a) 15
b) 22
c) 34
d) 45
e) 50

Answers

Answer:

1,3,5,15

1,2,11,22

1,2,17,34

1,3,5,9,15,45

1,2,5,10,25,50

Step-by-step explanation:

Hope it will help u

y=-4x - 8
6x + 2y=-10


I need the equation solved by substitution

Please show work :)

Answers

Answer:

x =1.28571428571

Step-by-step explanation:

6x + 2y = -10

y = -4x - 8

6x + 2(4x - 8) = -10

6x + 8x - 8 = -10

14x - 8 = 10

(we put together the like terms)

14x = 10 + 8

14x = 18

x =  [tex]\frac{18}{14}[/tex]

x = 1.28571428571

Simple geometry equation

Answers

Answer:

106 or 4x+18  

Step-by-step explanation:

Since Ray QR is a bisector you can determine that PRQ and QRS are equal angles (def. of a bisector). Then, what I chose to do is graph them and find where they intercept which was 22. We now know 22 is x so we can substitute it into the equation and solve to deterine that our answer is 106.

22 = x, explanation:
x+ 31 = 3x-13
-x. -x
31=2x-13
+13.. +13
44-=2x
—. .—
2. 2
22= x

Use the table to complete the work to find the missing value. Conversion Chart Pints Ounces 3 48 7 ? 30 480 3 pints 48 ounces 11 pints ? ounces How many ounces are in 11 pints? o 144 o160 o 176 o 192​

Answers

Answer:

176 ounces (C)

Step-by-step explanation:

if you divide 48 by 3 it will give you 16 and there are 11 pints so u wanna multiply 16 and 11 to 176 ounces..

also i got it right on edge :P

Answer: 176

Step-by-step explanation:

You borrow $350 from your aunt and agree to repay her $400 ($350 principal + $50 interest) in 18 months. What interest rate (using simple interest, and to the nearest tenth) are you paying?

Answers

Answer:

The interest rate is 0.79% per month.

Step-by-step explanation:

Present value of borrowed amount = $350

Future value of borrowed amount = $400

Interest amount = $50

Time duration = 18 months or 1.5 years

Now we have to find the rate of interest by using the above information. Here below is the calculation of interest rate.

Interest rate =  P×R×T

50 = 350 × R×18

R = 0.007936

Or, R = 0.79% per month.

What is the smallest positive number that is prime and 10 less than a perfect square?

Answers

Answer:

The problem states that the answer cannot be a perfect square or have prime factors less than $50$. Therefore, the answer will be the product of at least two different primes greater than $50$. The two smallest primes greater than $50$ are $53$ and $59$. Multiplying these two primes, we obtain the number $3127$, which is also the smallest number on the list of answer choices. So we are done, and the answer is $\boxed{\textbf{(A)}\ 3127}$.

Step-by-step explanation:

Answer:

71

Step-by-step explanation:

I just did the AOPS question, you can see the attachment down below.

Hope this helped! :)

Other Questions
The contagent of 180/3,180/4,180/6 is If (6, 1) is dilated by a scale factor of 3 the new point would be:A(3,3)B (18,3)C (15,3)D (9.4) A football coach walks 18 meters westward, then 12 meterseastward, then 28 meters westward, and finally 14 meterseastward. A bag contains three types of candy (gumdrops, jelly beans, and jawbreakers). Ifchosen at random, the probability of getting a gumdrop is, and the probabilityof getting a jellybean is 3. What is the probability of getting a jawbreaker? What is the quotient of 8,688 24? 362 434 450 8,664 PT=4x+3 and TQ=8x-9 Find the value of PT Consider the figure above. Starting with the shape in the upper left, identify the figure that has been: -Translated only. -Rotated and translated. -Reflected about the x-axis. Solving by Completing the Square a pencil is 7 in Long what is its length in centimeters What is the difference between bulk and particle scale? Someone please help!! Drag the tiles to the correct boxes to complete the pairs. Match the different financial ratios with their one example of these ratios. which of these is a benefit of social networking? Which of the following explains the relationship between angles a and b? Lines JK and JM intersect at point J, creating four angles identified as angle B, angle D, angle C, and angle a clockwise from the top right. Adjacent angles, corresponding notes, Complementary angles or vertival angles india experienced a golden age under the gupta empire. which statement describes this period? James usually___to work by car,but today he____ the bus. Type the correct answer in each box. Use numerals instead of words. If necessary, use / for the fraction bar(s). Points A and B are the endpoints of an arc of a circle. Chords are drawn from the two endpoints to a third point, C, on the circle. Given m AB =64 and ABC=73 , mACB=....... and mAC=.... i will give brainliest Which of the following structures are present in plant cells but absent in animal cells? Choose 1 answer: (Choice A) A Cell walls (Choice B) B Mitochondria (Choice C) C Cytosol (Choice D) D Golgi bodies Which of the following is NOT a part of cell theory a.All living things are made of one or more cells. b.All cells are Eukaryotic cells. c.Cells are the basic unit of life. d.Cells come from pre-existing cells. ___KClO3--->___KCl+___O2 I need help bro. PLEASE HELP!! I worked it out and havent found the right answer